How Do You Calculate the Expected Value of Geometric Brownian Motion?

Click For Summary
SUMMARY

The discussion focuses on calculating the expected value of a geometric Brownian motion, denoted as S(t), with initial value S(0) = S_0 and parameters μ and σ². The participants derive an approximation for S(t) using random variables Z_i, which represent increments of Brownian motion. The expected value E[S(t)] is shown to converge to S_0 * exp(tμ + (tσ²)/2) as the limit of the expectation is taken. The discussion emphasizes the importance of using the limit property lim (1+x/t)^t = exp(x) to finalize the calculation.

PREREQUISITES
  • Understanding of geometric Brownian motion and its properties.
  • Familiarity with stochastic calculus and Brownian motion.
  • Knowledge of limits and exponential functions in probability theory.
  • Basic concepts of random variables and their distributions.
NEXT STEPS
  • Study the derivation of the expected value for geometric Brownian motion in detail.
  • Learn about stochastic calculus applications in finance, particularly in option pricing models.
  • Explore the properties of the log-normal distribution as it relates to geometric Brownian motion.
  • Investigate the use of the Central Limit Theorem in the context of sums of random variables.
USEFUL FOR

Mathematicians, financial analysts, and data scientists interested in stochastic processes and their applications in finance, particularly those working with models involving geometric Brownian motion.

motherh
Messages
27
Reaction score
0
Hi, I am trying to answer the following question:

Consider a geometric Brownian motion S(t) with S(0) = S_0 and parameters μ and σ^2. Write down an approximation of S(t) in terms of a product of random variables. By taking the limit of the expectation of these compute the expectation of S(t).

Let Z_i = σ√Δ (with probability 1/2(1+ \frac{μ}{σ}√Δ)) or -σ√Δ (with probability 1/2(1- \frac{μ}{σ}√Δ)).

Then logS(t) ≈ Z_1 + ... + Z_(\frac{t}{Δ}) (as logS(t) is Brownian motion)

And this approximation gets better as \frac{t}{Δ} tends to infinity

S(t) ≈ exp(Z_1) * ... * exp(Z_(\frac{t}{Δ}))

So now exp(Z_i) = exp(σ√Δ) (with probability 1/2(1+ \frac{μ}{σ}√Δ)) or exp(-σ√Δ) (with probability 1/2(1- \frac{μ}{σ}√Δ)).

E[S(t)] ≈ (E[exp(Z_1)])^\frac{t}{Δ}

I know that the answer I am looking for is S_0*exp(tμ+\frac{tσ^2}{2}). Also

E[exp(Z_1)] =1/2exp(σ√Δ)*(1+ \frac{μ}{σ}√Δ) -1/2exp(-σ√Δ)*(1- \frac{μ}{σ}√Δ).

So I believe that

E[S(t)] = lim [ 1/2exp(σ√Δ)*(1+ \frac{μ}{σ}√Δ) -1/2exp(-σ√Δ)*(1- \frac{μ}{σ}√Δ) ]^\frac{t}{Δ} (as \frac{t}{Δ} tends to infinity)

Where do I go from here? I want to make use of lim (1+x/t)^t = exp(x) but how do I go from here? Help is MUCH appreciated!
 
Physics news on Phys.org
motherh said:
Hi, I am trying to answer the following question:

Consider a geometric Brownian motion S(t) with S(0) = S_0 and parameters μ and σ^2. Write down an approximation of S(t) in terms of a product of random variables. By taking the limit of the expectation of these compute the expectation of S(t).

Let Z_i = σ√Δ (with probability 1/2(1+ \frac{μ}{σ}√Δ)) or -σ√Δ (with probability 1/2(1- \frac{μ}{σ}√Δ)).

Then logS(t) ≈ Z_1 + ... + Z_(\frac{t}{Δ}) (as logS(t) is Brownian motion)

And this approximation gets better as \frac{t}{Δ} tends to infinity

S(t) ≈ exp(Z_1) * ... * exp(Z_(\frac{t}{Δ}))

So now exp(Z_i) = exp(σ√Δ) (with probability 1/2(1+ \frac{μ}{σ}√Δ)) or exp(-σ√Δ) (with probability 1/2(1- \frac{μ}{σ}√Δ)).

E[S(t)] ≈ (E[exp(Z_1)])^\frac{t}{Δ}

I know that the answer I am looking for is S_0*exp(tμ+\frac{tσ^2}{2}). Also

E[exp(Z_1)] =1/2exp(σ√Δ)*(1+ \frac{μ}{σ}√Δ) -1/2exp(-σ√Δ)*(1- \frac{μ}{σ}√Δ).

So I believe that

E[S(t)] = lim [ 1/2exp(σ√Δ)*(1+ \frac{μ}{σ}√Δ) -1/2exp(-σ√Δ)*(1- \frac{μ}{σ}√Δ) ]^\frac{t}{Δ} (as \frac{t}{Δ} tends to infinity)

Where do I go from here? I want to make use of lim (1+x/t)^t = exp(x) but how do I go from here? Help is MUCH appreciated!


Use standard probability results to figure out what is the distribution of ##\sum_i Z_i## in the limit as ##n \to \infty, \Delta \to 0## with ##t = n \Delta## fixed.
 
Last edited:
Question: A clock's minute hand has length 4 and its hour hand has length 3. What is the distance between the tips at the moment when it is increasing most rapidly?(Putnam Exam Question) Answer: Making assumption that both the hands moves at constant angular velocities, the answer is ## \sqrt{7} .## But don't you think this assumption is somewhat doubtful and wrong?

Similar threads

  • · Replies 7 ·
Replies
7
Views
2K
Replies
5
Views
2K
  • · Replies 4 ·
Replies
4
Views
2K
Replies
4
Views
3K
  • · Replies 1 ·
Replies
1
Views
5K
Replies
7
Views
3K
Replies
1
Views
1K
Replies
2
Views
5K
Replies
5
Views
2K
Replies
1
Views
2K